*NURSING > TEST BANK > McCance, Huether - Pathophysiology_ The Biological Basis for Disease in Adults and Children TEST BAN (All)

McCance, Huether - Pathophysiology_ The Biological Basis for Disease in Adults and Children TEST BANK. 8th-Mosby ( ALL ANSWERS CORRECT) COVERS CHAPTER 1 - 50

Document Content and Description Below

Chapter 1: Cellular Biology MULTIPLE CHOICE 1. Which statement best describes the cellular function of metabolic absorption? a. Cells can produce proteins. c. Cells can take in and use nutrients. ... b. Cells can secrete digestive enzymes. d. Cells can synthesize fats. ANS: C In metabolic absorption, all cells take in and use nutrients and other substances from their surroundings. The remaining options are not inclusive in their descriptions of cellular metabolic absorption. PTS: 1 REF: Page 2 2. Most of a cell’s genetic information, including RNA and DNA, is contained in the: a. Mitochondria c. Nucleolus b. Ribosome d. Lysosome ANS: C The nucleus contains the nucleolus, a small dense structure composed largely of RNA, most of the cellular DNA, and the DNA-binding proteins, such as the histones, which regulate its activity. The other options do not contain most of a cell’s genetic information. PTS: 1 REF: Page 2 3. Which component of the cell produces hydrogen peroxide (H2O2) by using oxygen to remove hydrogen atoms from specific substrates in an oxidative reaction? a. Lysosomes c. Ribosomes b. Peroxisomes d. Oxyhydrosomes ANS: B Peroxisomes are so named because they usually contain enzymes that use oxygen to remove hydrogen atoms from specific substrates in an oxidative reaction that produces H2O2, which is a powerful oxidant and potentially destructive if it accumulates or escapes from peroxisomes. Ribosomes are RNA-protein complexes (nucleoproteins) that are synthesized in the nucleolus and secreted into the cytoplasm through pores in the nuclear envelope called nuclear pore complexes. Lysosomes are saclike structures that originate from the Golgi complex and contain more than 40 digestive enzymes called hydrolases, which catalyze bonds in proteins, lipids, nucleic acids, and carbohydrates. Oxyhydrosomes are involved in enzyme production. PTS: 1 REF: Page 8 4. Which cell component is capable of cellular autodigestion when it is released during cell injury? a. Ribosome c. Smooth endoplasmic reticulum b. Golgi complex d. Lysosomes ANS: D NURSINGTB.COM PATHOPHYSIOLOGY 8TH EDITION MCCANCE TEST BANKNURSINGTB.COM The lysosomal membrane acts as a protective shield between the powerful digestive enzymes within the lysosome and the cytoplasm, preventing their leakage into the cytoplasmic matrix. Disruption of the membrane by various treatments or cellular injury leads to a release of the lysosomal enzymes, which can then react with their specific substrates, causing cellular self-digestion. The other options do not correctly describe this process. PTS: 1 REF: Pages 7-8 5. What is the sequence of steps in the development of a digestive enzyme by the pancreas cells from the initial transcription to the release from the cell? a. The enzyme is transcribed from DNA by RNA in the nucleus, proceeds to the ribosome for synthesis, and is transported in a secretory vesicle to the cell membrane. b. The enzyme is transcribed from RNA by DNA in the nucleus, proceeds to the lysosome for synthesis, and is transported in an encapsulated membrane to the cell membrane. c. The enzyme is transcribed by the mitochondria in the nucleus, proceeds to the ribosome for synthesis, and is transported in a cytoskeleton to the cell membrane. d. The enzyme is transcribed from DNA by RNA in the nucleus, proceeds to the Golgi complex for synthesis, and is transported in a cytosol to the cell membrane. ANS: A The enzyme is transcribed from DNA by RNA in the nucleus, proceeds to the ribosome for synthesis, and is transported in a secretory vesicle to the cell membrane. The other options do not correctly describe this process. PTS: 1 REF: Page 7 | Figure 1-5 6. During which phase of the cell cycle is DNA synthesized? a. G1 c. G2 b. S d. M ANS: B The four designated phases of the cell cycle are: (1) the G1 phase (G = gap), which is the period between the M phase (M = mitosis) and the start of DNA synthesis; (2) the S phase (S = synthesis), during which DNA is synthesized in the cell nucleus; (3) the G2 phase, during which RNA and protein synthesis occurs, the period between the completion of DNA synthesis and the next phase (M); and (4) the M phase, which includes nuclear and cytoplasmic division. PTS: 1 REF: Page 37 7. What organic compound facilitates transportation across cell membranes by acting as receptors, transport channels for electrolytes, and enzymes to drive active pumps? a. Lipids c. Proteins b. Proteases d. Carbohydrates ANS: C NURSINGTB.COM PATHOPHYSIOLOGY 8TH EDITION MCCANCE TEST BANKNURSINGTB.COM Proteins act as (1) recognition and binding units (receptors) for substances moving in and out of the cell; (2) pores or transport channels for various electrically charged particles called ions or electrolytes and specific carriers for amino acids and monosaccharides; and (3) specific enzymes that drive active pumps that promote the concentration of certain ions, particularly potassium (K+), within the cell while keeping concentrations of other ions, for example, sodium (Na+), below the concentrations found in the extracellular environment. The other options do not correctly describe this process. PTS: 1 REF: Page 13 | Page 15 8. Understanding the various steps of proteolytic cascades, such as caspase-mediated apoptosis and complement cascades, may be useful in designing drug therapy for which human diseases? a. Cardiac and vascular disorders b. Autoimmune and malignant disorders c. Gastrointestinal and renal disorders d. Endocrine and gastrointestinal disorders ANS: B Understanding the various steps involved in this process is crucial for designing drug interventions. Dysregulation of proteases features prominently in many human diseases, including cancer, autoimmunity, and neurodegenerative disorders. The other options do not correctly describe this process. PTS: 1 REF: Page 15 9. Which structure prevents water-soluble molecules from entering cells across the plasma membrane? a. Carbohydrate chains c. Membrane channel proteins b. Glycoprotein channels d. Lipid bilayer ANS: D The bilayer’s structure accounts for one of the essential functions of the plasma membrane. It is impermeable to most water-soluble molecules (molecules that dissolve in water) because the water-soluble molecules are insoluble in the oily core region. The bilayer serves as a barrier to the diffusion of water and hydrophilic substances while allowing lipid-soluble molecules, such as oxygen (O2) and carbon dioxide (CO2), to diffuse through it readily. The other options do not correctly describe this process. PTS: 1 REF: Pages 12-13 10. The fluid mosaic model explains: a. How a cell membrane functions b. Why our bodies appear to be solid c. How tissue is differentiated d. How fluid moves between the intracellular and extracellular compartments ANS: A The fluid mosaic model accounts for the flexibility of cellular membranes, their self-sealing properties, and their impermeability to many substances. The remaining options do not explain the mosaic model. NURSINGTB.COM PATHOPHYSIOLOGY 8TH EDITION MCCANCE TEST BANKNURSINGTB.COM PTS: 1 REF: Page 12 | What's New box 11. Which form of cell communication is used to communicate within the cell itself and with other cells in direct physical contact? a. Protein channel (gap junction) b. Plasma membrane–bound signaling molecules (involving receptors) c. Hormone secretion such as neurotransmitters d. Extracellular chemical messengers such as ligands ANS: A Cells communicate by using hundreds of kinds of signal molecules, for example, insulin. Cells communicate in three main ways; they display plasma membrane–bound signaling molecules (receptors) that affect the cell itself and other cells in direct physical contact. The other options do not correctly describe this process. PTS: 1 REF: Page 20 12. Which mode of chemical signaling uses blood to transport communication to cells some distance away? a. Paracrine c. Neurotransmitter b. Autocrine d. Hormonal ANS: D Chemical signaling can be classified into three categories: (1) local-chemical mediator, (2) hormone, and (3) neurotransmitter. In the local-chemical mediator model, the secreted chemical acts on the cells in the immediate environment. Hormones are used for communication with distant target cells. For example, cells can secrete a chemical and rely on the blood system to deliver the signal to a distant cell. Finally, neurotransmitters are secreted by neurons to stimulate an adjoining cell. For example, a neuron might secrete acetylcholine to stimulate the movement of a muscle cell. PTS: 1 REF: Page 20 13. Which mode of chemical signaling uses local chemical mediators that are quickly taken up, destroyed, or immobilized? a. Paracrine c. Neurotransmitter b. Autocrine d. Hormone ANS: A In paracrine signaling, cells secrete local chemical mediators that are quickly taken up, destroyed, or immobilized. The other options do not correctly describe this process. PTS: 1 REF: Page 20 14. Neurotransmitters affect the postsynaptic membrane by binding to: a. Lipids c. Amphipathic lipids b. Ribosomes d. Receptors ANS: D In each type of chemical signaling, the target cell receives the signal by first attaching to its receptors. The other options do not correctly describe this process. NURSINGTB.COM PATHOPHYSIOLOGY 8TH EDITION MCCANCE TEST BANKNURSINGTB.COM PTS: 1 REF: Page 17 15. How do cells receive communication from the extracellular fluid surrounding them? a. Protein channel (gap junction) b. Plasma membrane–bound signaling molecules (involving receptors) c. Hormone secretion such as neurotransmitters d. Chemical messengers such as ligands ANS: D Cellular communication can occur by the binding of a chemical massager (a ligand) to a specific membrane receptor that is closely associated with the channel (e.g., G proteins). The other options do not correctly describe how cells communicate. PTS: 1 REF: Pages 21-22 16. When a second message is necessary for extracellular communication to be activated, it is provided by which one? a. Guanosine triphosphate (GTP) c. Adenosine triphosphate (ATP) b. Adenosine monophosphate (AMP) d. Guanosine diphosphate (GDP) ANS: B The two major second messenger pathways are cyclic AMP (cAMP) and calcium (Ca++). PTS: 1 REF: Pages 22-23 17. Under anaerobic conditions, what process provides energy for the cell? a. Oxidative phosphorylation c. Lactolysis b. Glycolysis d. Passive transport ANS: B Glycolysis produces a net of two molecules of ATP per glucose molecule through the process of oxidation or the removal and transfer of a pair of electrons. The other options do not correctly identify an anaerobic process that provides energy to the cell. PTS: 1 REF: Page 28 18. What is the mechanism by which the energy produced from carbohydrates, proteins, and lipids is transferred to adenosine triphosphate (ATP)? a. Anaerobic glycolysis c. Oxidative phosphorylation b. Oxidative cellular metabolism d. Tricarboxylic acid phosphorylation ANS: C Oxidative phosphorylation occurs in the mitochondria and is the mechanism by which the energy produced from carbohydrates, fats, and proteins is transferred to ATP. The other options do not correctly identify the mechanism described in the question. PTS: 1 REF: Pages 27-28 19. Passive transport is best described with which statement? a. Being driven by osmosis, hydrostatic pressure, and diffusion b. Involving receptors that can bind with substances being transported NURSINGTB.COM PATHOPHYSIOLOGY 8TH EDITION MCCANCE TEST BANKNURSINGTB.COM c. Being capable of transporting macromolecules d. Requiring energy generated by the cell ANS: A Water and small electrically uncharged molecules move easily through pores in the plasma membrane’s lipid bilayer. This process, called passive transport, naturally occurs through any semipermeable barrier. It is driven by osmosis, hydrostatic pressure, and diffusion, all of which depend on the laws of physics and do not require life. The other options do not correctly describe passive transport. PTS: 1 REF: Page 28 20. Active transport occurs across which type of membranes? a. Membranes that have a higher concentration of the solute on the outside of the cell b. Membranes that are semipermeable to water and small electrically uncharged molecules c. Membranes that have receptors that are capable of binding with the substances to be transported d. Membranes that have a cell membrane that is hydrophobic rather than hydrophilic ANS: C Some molecules are moved into the cell by mechanisms of active transport, which require receptors that are capable of recognizing and binding with the substance to be transported. Diffusion is the movement of a solute molecule from an area of greater solute concentration to an area of lesser solute concentration. Hydrostatic pressure is the mechanical force of water pushing against cellular membranes. Osmosis is the movement of water down a concentration gradient; that is, across a semipermeable membrane from a region of higher water concentration to a lower water concentration PTS: 1 REF: Page 28 21. Which method of transport uses transmembrane proteins with receptors with a high degree of specificity for the substance being transported? a. Active c. Transmembranous b. Mediated d. Passive ANS: B Mediated transport (passive and active) involves integral or transmembrane proteins with receptors having a high degree of specificity for the substance being transported. Inorganic anions and cations (e.g., Na+, K+, Ca++, chloride [Cl–], bicarbonate [HCO3–]) and charged and uncharged organic compounds (e.g., amino acids, sugars) require specific transport systems to facilitate movement through different cellular membranes. The remaining options do not correctly identify the process described. PTS: 1 REF: Page 31 22. The movement of fluid across the arterial end of capillary membranes into the interstitial fluid surrounding the capillary is an example of which fluid movement process? a. Hydrostatic pressure c. Diffusion b. Osmosis d. Active transport ANS: A NURSINGTB.COM PATHOPHYSIOLOGY 8TH EDITION MCCANCE TEST BANKNURSINGTB.COM Hydrostatic pressure is the mechanical force of water pushing against cellular membranes. In the vascular system, hydrostatic pressure is the blood pressure generated in vessels by the contraction of the heart. Blood reaching the capillary bed has a hydrostatic pressure of 25 to 30 mm Hg, which is sufficient force to push water across the thin capillary membranes into the interstitial space. The remaining options do not correctly identify the process described. PTS: 1 REF: Pages 29-30 23. Why is osmolality preferred over osmolarity as the measurement of osmotic activity in the clinical assessment of individuals? a. Plasma contains sodium and chloride, which influence the volume of solution. b. Volume affects perfusion more than the weight of solutes. c. More of the weight of plasma is influenced by solutes, such as protein and glucose, rather than by water. d. Osmotic activity depends on the concentration of solutes present in plasma, such as proteins and glucose. ANS: C In plasma, less of the plasma weight is water; therefore the overall concentration of particles is greater. The osmolality will be greater than the osmolarity because of the smaller proportion of water. Osmolality is thus the preferred measure of osmotic activity in clinical assessment of individuals. PTS: 1 REF: Page 30 24. A patient who has diarrhea receives a 3% saline solution intravenously to replace the sodium and chloride lost in the stool. What effect will this fluid replacement have on cells? a. Become hydrated c. Shrink b. Swell or burst d. Divide ANS: C A hypertonic solution has a concentration of greater than 285 to 294 mOsm/kg. An example of a hypertonic solution is 3% saline solution. Water can be pulled out of the cells by a hypertonic solution; therefore the cells shrink. The remaining options do not correctly describe the effect identified in the stem. PTS: 1 REF: Page 31 25. The transport of glucose from the blood to the cell is accomplished by which process? a. Active-mediated transport (active transport) b. Active diffusion c. Passive osmosis d. Passive-mediated transport (facilitated diffusion) ANS: D Facilitated diffusion is the means by which glucose is transported from the blood to the cells. The remaining options do not correctly identify this process. PTS: 1 REF: Pages 31-32 26. Potassium and sodium are transported across plasma membranes by: NURSINGTB.COM PATHOPHYSIOLOGY 8TH EDITION MCCANCE TEST BANKNURSINGTB.COM a. Passive electrolyte channels b. Coupled channels c. Adenosine triphosphatase (ATPase) enzyme d. Diffusion ANS: C The exact mechanism for the transport of Na+ and K+ across the membrane is uncertain. One proposal is that ATPase enzyme induces the transporter protein to undergo several conformational changes, causing Na+ and K+ to move short distances (see Figure 1-29). The remaining options do not correctly describe the means by which K+ and Na+ are transported. PTS: 1 REF: Pages 32-33 27. What occurs during exocytosis? a. Macromolecules can be secreted across eukaryotic cell membranes. b. All substances are secreted into the cellular matrix. c. No repairs in the plasma membrane can take place. d. Solute molecules flow freely into and out of the cell. ANS: A In eukaryotic cells, secretion of macromolecules almost always occurs by exocytosis. The remaining options do not correctly describe exocytosis. PTS: 1 REF: Pages 35-36 28. Why is it possible for potassium to diffuse easily into and out of cells? a. Potassium has a greater concentration in the intracellular fluid (ICF). b. Sodium has a greater concentration in the extracellular fluid (ECF). c. The resting plasma membrane is more permeable to potassium. d. An excess of anions are inside the cell. ANS: C Because the resting plasma membrane is more permeable to K+ than to Na+, K+ can easily diffuse from its area of higher concentration in the ICF to its area of lower concentration in the ECF. Because Na+ and K+ are both cations, the net result is an excess of anions inside the cell, resulting in the resting membrane potential. The remaining options do not correctly identify the process that most easily diffuses K+. PTS: 1 REF: Page 36 29. The cellular uptake of the nutrient cholesterol depends on which process? a. Receptor-mediated exocytosis c. Receptor-mediated endocytosis b. Antiport system d. Passive transport ANS: C The cellular uptake of nutrients, such as cholesterol, for example, depends on receptor-mediated endocytosis. Nutrients are not transported via the other options. PTS: 1 REF: Page 33 NURSINGTB.COM PATHOPHYSIOLOGY 8TH EDITION MCCANCE TEST BANKNURSINGTB.COM 30. What causes the rapid change in the resting membrane potential to initiate an action potential? a. Potassium gates open, and potassium rushes into the cell, changing the membrane potential from negative to positive. b. Sodium gates open, and sodium rushes into the cell, changing the membrane potential from negative to positive. c. Sodium gates close, allowing potassium into the cell to change the membrane potential from positive to negative. d. Potassium gates close, allowing sodium into the cell to change the membrane potential from positive to negative. ANS: B When a resting cell is stimulated through voltage-regulated channels, the cell membranes become more permeable to Na+. A net Na+ moves into the cell, and the membrane potential decreases, or moves forward, from a negative value (in millivolts) to zero. The Na+ gates open, and Na+ rushes into the cell, causing the membrane potential to reduce to zero and then become positive (depolarization). The remaining options do not correctly describe the change that initiates an action potential. PTS: 1 REF: Page 36 31. The action of platelet-derived growth factor is to stimulate the production of which cells? a. Platelets c. Connective tissue cells b. Epidermal cells d. Fibroblast cells ANS: C Different types of cells require different factors; for example, platelet-derived growth factor stimulates the production of connective tissue cells. The remaining options do not correctly describe the action of platelet-derived growth factor. PTS: 1 REF: Page 39 32. The role of cytokines in cell reproduction is that they: a. Provide growth factor for tissue growth and development. b. Block progress of cell reproduction through the cell cycle. c. Restrain cell growth and development. d. Provide nutrients for cell growth and development. ANS: A Growth factors, also called cytokines, are peptides that transmit signals within and among cells. They have a major role in the regulation of tissue growth and development (see Table 1-6). The remaining options do not correctly describe the role of cytokines in cell reproduction. PTS: 1 REF: Pages 38-39 33. What is the process of cellular reproduction? a. The process often takes months or years to complete. b. Cellular reproduction typically has a short interphase. c. Two diploid cells, called daughter cells, have been formed. d. The process involves the interaction of male and female cells. NURSINGTB.COM PATHOPHYSIOLOGY 8TH EDITION MCCANCE TEST BANKNURSINGTB.COM ANS: C During telophase, the final stage, a new nuclear membrane is formed around each group of 46 chromosomes, the spindle fibers disappear, and the chromosomes begin to uncoil. Cytokinesis causes the cytoplasm to divide into roughly equal parts during this phase. At the end of telophase, two identical diploid cells, called daughter cells, have been formed from the original cell. PTS: 1 REF: Page 37 34. Which statement is true about eukaryotic cells? a. They lack distinct nucleus. b. They contain compartments called organelles. c. They lack an encasing nuclear membrane. d. They are smaller than the typical prokaryote cell. ANS: B Eukaryotic cells have a characteristic set of membrane-bound intracellular compartments called organelles that include a well-defined nucleus and are larger than prokaryotes. The remaining statements are not true regarding eukaryotic cells. PTS: 1 REF: Page 2 35. Which statement is true about phagocytosis? a. Phagocytosis is an example of exocytosis. b. Phagocytosis is dependent on small vesicles. c. Phagocytosis involves the ingestion of bacteria. d. Phagocytosis focuses on solute molecules. ANS: C In phagocytosis, the large molecular substances are engulfed by the plasma membrane and enter the cell so that they can be isolated and destroyed by lysosomal enzymes. Two types of endocytosis are designated, based on the size of the vesicle formed. Pinocytosis (cell drinking) involves the ingestion of fluids and solute molecules through the formation of small vesicles, and phagocytosis (cell eating) involves the ingestion of large particles, such as bacteria, through formation of large vesicles (also called vacuoles). Phagocytosis in an example of endocytosis, not exocytosis. PTS: 1 REF: Pages 33-34 36. A muscle cell possesses which specialized function? a. Movement c. Secretion b. Conductivity d. Respiration ANS: A A cell has the potential to differentiation and to gain the ability to perform one of eight specialized functions. Muscle cells can generate forces that produce motion. Nerves cells are capable of conductivity. Cells of the adrenal gland, testis, and ovary can secrete. Respiration is a function that all cells possess. PTS: 1 REF: Page 2 NURSINGTB.COM PATHOPHYSIOLOGY 8TH EDITION MCCANCE TEST BANKNURSINGTB.COM 37. When a mucous gland cell creates a new substance from previously absorbed material, this process is known as which specialized cellular function? a. Excretion c. Reproduction b. Metabolic absorption d. Secretion ANS: D Certain cells, such as mucous gland cells, can synthesize new substances from substances they absorb and then secrete the new substances to serve elsewhere as needed. The other options are not used to describe the function described in the stem. PTS: 1 REF: Page 2 38. All cells are capable of what process? a. Excretion c. Metabolic absorption b. Movement d. Continuous division ANS: A All cells have the capacity to excrete, thus allowing them to rid themselves of waste products resulting from the metabolic breakdown of nutrients. The remaining options are not functions possessed by all cells. PTS: 1 REF: Page 2 MULTIPLE RESPONSE 39. What are the major chemical components of the cell membranes? (Select all that apply.) a. Lipids b. Sodium ions c. Carbohydrates d. DNA e. Proteins ANS: A, E The major chemical components of all cell membranes are lipids and proteins, but the percentage of each varies among different membranes. PTS: 1 REF: Page 12 40. Which cells lose their ability to replicate and divide? (Select all that apply.) a. Intestines b. Nerves c. Skin d. Lens of the eye e. Skeletal muscle ANS: B, D, E All types of cells undergo mitosis during the formation of the embryo, but many adult cells, such as nerve cells, lens cells of the eye, and muscle cells, lose their ability to replicate and divide. Intestines and skin cells retain their ability to replicate and divide. PTS: 1 REF: Page 37 NURSINGTB.COM PATHOPHYSIOLOGY 8TH EDITION MCCANCE TEST BANKNURSINGTB.COM 41. Which statements are true concerning the process of facilitated diffusion? (Select all that apply). a. Facilitated diffusion is also referred to as passive mediated transport. b. This process expends no metabolic energy. c. Moving solute molecules through cellular membranes are involved in this process. d. Movement up a concentration gradient is necessary. e. Facilitated diffusion is the primary means for water transport. ANS: A, B, C In passive mediated transport, also called facilitated diffusion, the protein transporter moves solute molecules through cellular membranes without expending metabolic energy. Downward movement along a concentration gradient is necessary. Osmosis is the movement of water down a concentration gradient. PTS: 1 REF: Pages 31-32 42. Passive transport is dependent on: (Select all that apply.) a. Semipermeable barrier membrane b. The process of osmosis c. Diffusion as a driving force d. A living host e. Hydrostatic pressure ANS: A, B, C, E Passive transport naturally occurs through any semipermeable barrier. It is driven by osmosis, hydrostatic pressure, and diffusion, all of which depend on the laws of physics and do not require life. PTS: 1 REF: Page 28 43. What is the primary function of proteins? (Select all that apply.) a. Proteins are binding units. b. Proteins are transport channels. c. Proteins are ribonucleoproteins. d. Proteins provide cell surface markers. e. Proteins are chemical reaction catalysts. ANS: A, B, D, E Protein functions include (a) recognition and binding units (receptors) for substances moving in and out of the cell; (b) pores or transport channels; (c) enzymes that drive active pumps; (d) cell surface markers, such as glycoproteins; (e) cell adhesion molecules; and (f) catalysts of chemical reactions. PTS: 1 REF: Page 15 MATCHING Match the structure with its function. Answers may be used more than once. ______ A. Endoplasmic reticulum ______ B. Ribosome ______ C. Secretory vesicle NURSINGTB.COM PATHOPHYSIOLOGY 8TH EDITION MCCANCE TEST BANKNURSINGTB.COM ______ D. Lysosomes 44. Packages and transports proteins. 45. Fuses with the plasma membrane to release contents from the cell. 46. Synthesizes and transports lipids. 47. Provides energy to digest proteins into amino acids. 44. ANS: A PTS: 1 REF: Pages 5-6 MSC: The endoplasmic reticulum (endo = within; plasma = cytoplasm; reticulum = network) is a membrane factory that specializes in the synthesis and transport of the protein and lipid components of most of the cell's organelles. 45. ANS: C PTS: 1 REF: Pages 6-7 MSC: Proteins from the endoplasmic reticulum are processed and packaged into small membrane-bound sacs or vesicles called secretory vesicles that collect at the end of the membranous folds of the Golgi bodies. The secretory vesicles then break off from the Golgi complex and migrate to a variety of intracellular and extracellular destinations, including the plasma membrane. The vesicles fuse with the plasma membrane, and their contents are released from the cell. 46. ANS: A PTS: 1 REF: Pages 5-6 MSC: The endoplasmic reticulum (endo = within; plasma = cytoplasm; reticulum = network) is a membrane factory that specializes in the synthesis and transport of the protein and lipid components of most of the cell's organelles. 47. ANS: D PTS: 1 REF: Page 7 MSC: Lysosomes function as the intracellular digestive system (see Figure 1-6). Lysosomal enzymes are capable of digesting most cellular constituents down to their basic forms, such as amino acids, fatty acids, and sugars. Match the structure with its function. Answers may be used more than once. ______ A. Passive-mediated transport ______ B. Active-mediated transport ______ C. Osmosis 48. Movement of water 49. Protein carrier 50. Facilitated diffusion 48. ANS: C PTS: 1 REF: Page 30 MSC: Osmosis is the movement of water down a concentration gradient; that is, across a semipermeable membrane from a region of higher water concentration to a lower water concentration. 49. ANS: B PTS: 1 REF: Page 32 MSC: In active-mediated transport, also called active transport, the protein transporter moves molecules against, or up, the concentration gradient. Unlike passive-mediated transport, active-mediated transport requires the expenditure of energy. 50. ANS: A PTS: 1 REF: Pages 31-32 MSC: In passive-mediated transport, also called facilitated diffusion, the protein transporter moves solute molecules through cellular membranes without expending metabolic energy. NURSINGTB.COM PATHOPHYSIOLOGY 8TH EDITION MCCANCE TEST BANKNURSINGTB.COM Chapter 2: Altered Cellular and Tissue Biology MULTIPLE CHOICE 1. Which type of cell adaptation occurs when normal columnar ciliated epithelial cells of the bronchial lining have been replaced by stratified squamous epithelial cells? a. Hyperplasia c. Dysplasia b. Metaplasia d. Anaplasia ANS: B Metaplasia is the reversible replacement of one mature cell by another, sometimes a less differentiated cell type. The best example of metaplasia is the replacement of normal columnar ciliated epithelial cells of the bronchial (airway) lining by stratified squamous epithelial cells. The other options do not accurately describe the event in the question. PTS: 1 REF: Page 54 2. The loss of the adenosine triphosphate (ATP) during ischemia causes cells to: a. Shrink because of the influx of calcium (Ca). b. Shrink because of the influx of potassium chloride (KCl). c. Swell because of the influx of sodium chloride (NaCl). d. Swell because of the influx of nitric oxide (NO). ANS: C A reduction in ATP levels causes the plasma membrane’s sodium-potassium (Na+–K+) pump and sodium-calcium exchange to fail, which leads to an intracellular accumulation of sodium and calcium and diffusion of potassium out of the cell. (The Na+–K+ pump is discussed in Chapter 1.) Sodium and water can then freely enter the cell, and cellular swelling results. The other options do not accurately describe the result of ATP at the cellular level. PTS: 1 REF: Page 57 3. The mammary glands enlarge during pregnancy primarily as a consequence of hormonal: a. Atrophy c. Anaplasia b. Hyperplasia d. Dysplasia ANS: B Hormonal hyperplasia occurs chiefly in estrogen-dependent organs, such as the uterus and breast. The remaining options do not adequately describe the consequence of hormones on breast tissue during pregnancy. PTS: 1 REF: Page 53 4. Free radicals play a major role in the initiation and progression of which diseases? a. Cardiovascular diseases such as hypertension and ischemic heart disease b. Renal diseases such as acute tubular necrosis and glomerulonephritis c. Gastrointestinal diseases such as peptic ulcer disease and Crohn disease d. Muscular disease such as muscular dystrophy and fibromyalgia ANS: A NURSINGTB.COM PATHOPHYSIOLOGY 8TH EDITION MCCANCE TEST BANKNURSINGTB.COM Emerging data indicate that reactive oxygen species play major roles in the initiation and progression of cardiovascular alterations associated with hyperlipidemia, diabetes mellitus, hypertension, ischemic heart disease, and chronic heart failure. No current research connects the disorders mentioned in the other options to the effects of free radicals. PTS: 1 REF: Pages 59-60 5. Free radicals cause cell damage by: a. Stealing the cell’s oxygen to stabilize the electron, thus causing hypoxia b. Stimulating the release of lysosomal enzymes that digest the cell membranes c. Transferring one of its charged, stabilized atoms to the cell membrane, which causes lysis d. Giving up an electron, which causes injury to the chemical bonds of the cell membrane ANS: D A free radical is an electrically uncharged atom or group of atoms having an unpaired electron. Having one unpaired electron makes the molecule unstable; thus to stabilize, the molecule gives up an electron to another molecule or steals one. Therefore it is capable of forming injurious chemical bonds with proteins, lipids, or carbohydrates—key molecules in membranes and nucleic acids. The remaining options do not accurately describe the role played by free radicals in cell damage. PTS: 1 REF: Page 60 6. What is a consequence of plasma membrane damage to the mitochondria? a. Enzymatic digestion halts DNA synthesis. b. Influx of calcium ions halts ATP production. c. Edema from an influx in sodium causes a reduction in ATP production. d. Potassium shifts out of the mitochondria, which destroys the infrastructure. ANS: B The most serious consequence of plasma membrane damage is, as in hypoxic injury, to the mitochondria. An influx of calcium ions from the extracellular compartment activates multiple enzyme systems, resulting in cytoskeleton disruption, membrane damage, activation of inflammation, and eventually DNA degradation. Calcium ion accumulation in the mitochondria causes the mitochondria to swell, which is an occurrence that is associated with irreversible cellular injury. The injured mitochondria can no longer generate ATP, but they do continue to accumulate calcium ions. The remaining options do not accurately describe the consequence of plasma membrane damage to the mitochondria. PTS: 1 REF: Page 63 7. What is a consequence of leakage of lysosomal enzymes during chemical injury? a. Enzymatic digestion of the nucleus and nucleolus occurs, halting DNA synthesis. b. Influx of potassium ions into the mitochondria occurs, halting the ATP production. c. Edema of the Golgi body occurs, preventing the transport of proteins out of the cell. d. Shift of calcium out of the plasma membrane occurs, destroying the cytoskeleton. ANS: A NURSINGTB.COM PATHOPHYSIOLOGY 8TH EDITION MCCANCE TEST BANKNURSINGTB.COM Enzymatic digestion of cellular organelles, including the nucleus and nucleolus, ensues, halting the synthesis of DNA and ribonucleic acid (RNA). The remaining options do not accurately describe the consequence of lysosomal enzyme leakage during chemical injury. PTS: 1 REF: Page 63 8. Lead causes damage within the cell by interfering with the action of: a. Sodium and chloride c. Calcium b. Potassium d. ATP ANS: C Lead affects many different biologic activities at the cellular and molecular levels, many of which may be related to its ability to interfere with the functions of calcium. Lead does not appear to cause damage by interfering with the action of the other options. PTS: 1 REF: Page 66 9. Which statement is a description of the characteristics of apoptosis? a. Programmed cell death of scattered, single cells b. Characterized by swelling of the nucleus and cytoplasm c. Unpredictable patterns of cell death d. Results in benign malignancies ANS: A Apoptosis is an active process of cellular self-destruction, also known as programmed cell death, which is implicated in normal and pathologic tissue changes. The remaining options do not accurately describe the characteristics of apoptosis. PTS: 1 REF: Page 91 10. Lead poisoning affects the nervous system by: a. Interfering with the function of neurotransmitters b. Inhibiting the production of myelin around nerves c. Increasing the resting membrane potential d. Altering the transport of potassium into the nerves ANS: A Alterations in calcium may play a crucial role in the interference with neurotransmitters, which may cause hyperactive behavior and the proliferation of capillaries of the white matter and intercerebral arteries. The remaining options do not accurately describe the effects of lead poisoning of the nervous system. PTS: 1 REF: Page 66 11. Carbon monoxide causes tissue damage by: a. Competing with carbon dioxide so that it cannot be excreted b. Binding to hemoglobin so that it cannot carry oxygen c. Destroying the chemical bonds of hemoglobin so it cannot carry oxygen d. Removing iron from hemoglobin so it cannot carry oxygen ANS: B NURSINGTB.COM PATHOPHYSIOLOGY 8TH EDITION MCCANCE TEST BANKNURSINGTB.COM Because carbon monoxide’s affinity for hemoglobin is 200 times greater than that of oxygen, it quickly binds with the hemoglobin, preventing oxygen molecules from doing so. The remaining options do not accurately describe the means by which carbon monoxide damages tissue. PTS: 1 REF: Page 67 12. Acute alcoholism mainly affects which body system? a. Hepatic c. Renal b. Gastrointestinal d. Central nervous ANS: D Acute alcoholism mainly affects the central nervous system but may induce reversible hepatic and gastric changes. Other systems may evidentially be affected by chronic alcoholism. PTS: 1 REF: Page 68 13. During cell injury caused by hypoxia, an increase in the osmotic pressure occurs within the cell because: a. Plasma proteins enter the cell. b. The adenosine triphosphatase (ATPase)–driven pump is stronger during hypoxia. c. Sodium chloride enters the cell. d. An influx of glucose occurs through the injured cell membranes. ANS: C In hypoxic injury, movement of fluid and ions into the cell is associated with acute failure of metabolism and a loss of ATP production. Normally, the pump that transports sodium ions out of the cell is maintained by the presence of ATP and ATPase, the active-transport enzyme. In metabolic failure caused by hypoxia, reduced ATP and ATPase levels permit sodium to accumulate in the cell, whereas potassium diffuses outward. The increase of intracellular sodium increases osmotic pressure, which draws more water into the cell. (Transport mechanisms are described in Chapter 1.) The remaining options do not accurately describe the cell injury that results in increased osmotic pressure caused by hypoxia. PTS: 1 REF: Page 84 14. Which statement is true regarding the difference between subdural hematoma and epidural hematoma? a. No difference exists, and these terms may be correctly used interchangeably. b. A subdural hematoma occurs above the dura, whereas an epidural hematoma occurs under the dura. c. A subdural hematoma is often the result of shaken baby syndrome, whereas an epidural hematoma rapidly forms as a result of a skull fracture. d. A subdural hematoma usually forms from bleeding within the skull, such as an aneurysm eruption, whereas an epidural hematoma occurs from trauma outside the skull, such as a blunt force trauma. ANS: C NURSINGTB.COM PATHOPHYSIOLOGY 8TH EDITION MCCANCE TEST BANKNURSINGTB.COM A subdural hematoma is a collection of blood between the inner surface of the dura mater and the surface of the brain, resulting from the shearing of small veins that bridge the subdural space. Subdural hematomas can be the result of blows, falls, or sudden acceleration-deceleration of the head, which occurs in the shaken baby syndrome. An epidural hematoma is a collection of blood between the inner surface of the skull and the dura and is almost always associated with a skull fracture. The other options do not accurately describe the differences between the two hematomas. PTS: 1 REF: Page 72 | Table 2-6 15. What physiologic change occurs during heat exhaustion? a. Hemoconcentration occurs because of the loss of salt and water. b. Cramping of voluntary muscles occurs as a result of salt loss. c. Thermoregulation fails because of high core temperatures. d. Subcutaneous layers are damaged because of high core temperatures. ANS: A Heat exhaustion occurs when sufficient salt and water loss results in hemoconcentration. The other options do not accurately describe the physiologic changes that occur during heat exhaustion. PTS: 1 REF: Page 77 16. In hypoxic injury, sodium enters the cell and causes swelling because: a. The cell membrane permeability increases for sodium during periods of hypoxia. b. ATP is insufficient to maintain the pump that keeps sodium out of the cell. c. The lactic acid produced by the hypoxia binds with sodium in the cell. d. Sodium cannot be transported to the cell membrane during hypoxia. ANS: B In hypoxic injury, movement of fluid and ions into the cell is associated with acute failure of metabolism and a loss of ATP production. Normally, the presence of ATP and ATPase, the active-transport enzyme, maintains the pump that transports sodium ions out of the cell. In metabolic failure caused by hypoxia, reduced ATP and ATPase levels permit sodium to accumulate in the cell, whereas potassium diffuses outward. The other options do not accurately describe the cause of the swelling caused by hypoxia. PTS: 1 REF: Page 84 17. What is the most common site of lipid accumulation? a. Coronary arteries c. Liver b. Kidneys d. Subcutaneous tissue ANS: C Although lipids sometimes accumulate in heart and kidney cells, the most common site of intracellular lipid accumulation, or fatty change, is liver cells. Subcutaneous tissue is not a common site of lipid accumulation. PTS: 1 REF: Pages 84-85 18. What mechanisms occur in the liver cells as a result of lipid accumulation? a. Accumulation of lipids that obstruct the common bile duct, preventing flow of bile NURSINGTB.COM PATHOPHYSIOLOGY 8TH EDITION MCCANCE TEST BANKNURSINGTB.COM from the liver to the gallbladder b. Increased synthesis of triglycerides from fatty acids and decreased synthesis of apoproteins c. Increased binding of lipids with apoproteins to form lipoproteins d. Increased conversion of fatty acids to phospholipids ANS: B Lipid accumulation in liver cells occurs after cellular injury sets the following mechanisms in motion: increased synthesis of triglycerides from fatty acids (increases in the enzyme, D-glycerophosphatase, which can accelerate triglyceride synthesis) and decreased synthesis of apoproteins (lipid-acceptor proteins). The other options do not accurately describe this event. PTS: 1 REF: Pages 84-85 19. Hemoprotein accumulations are a result of the excessive storage of: a. Iron, which is transferred from the cells to the bloodstream b. Hemoglobin, which is transferred from the bloodstream to the cells c. Albumin, which is transferred from the cells to the bloodstream d. Amino acids, which are transferred from the cells to the bloodstream ANS: A Excessive storage of iron, which is transferred to the cells from the bloodstream, causes hemoprotein accumulations in cells. Hemoglobin, albumin, or amino acids will not cause hemoprotein accumulations. PTS: 1 REF: Page 86 20. Hemosiderosis is a condition that results in the excess of what substance being stored as hemosiderin in cells of many organs and tissues? a. Hemoglobin c. Iron b. Ferritin d. Transferrin ANS: C Hemosiderosis is a condition that occurs only when excess iron is stored as hemosiderin in the cells of many organs and tissues. PTS: 1 REF: Page 86 21. What is the cause of free calcium in the cytosol that damages cell membranes by uncontrolled enzyme activation? a. Activation of endonuclease interferes with the binding of calcium to protein. b. Activation of phospholipases, to which calcium normally binds, degrades the proteins. c. An influx of phosphate ions competes with calcium for binding to proteins. d. Depletion of ATP normally pumps calcium from the cell. ANS: D If abnormal direct damage occurs to membranes or ATP is depleted, then calcium increases in the cytosol. The other options do not accurately describe the cause of free calcium in cytosol to damage cell membranes. NURSINGTB.COM PATHOPHYSIOLOGY 8TH EDITION MCCANCE TEST BANKNURSINGTB.COM PTS: 1 REF: Pages 57-58 | Page 87 | Figure 2-24 22. What two types of hearing loss are associated with noise? a. Acoustic trauma and noise-induced c. High frequency and acoustic trauma b. High frequency and low frequency d. Noise-induced and low frequency ANS: A Two types of hearing loss are associated with noise: (1) acoustic trauma or instantaneous damage caused by a single sharply rising wave of sound (e.g., gunfire), and (2) noise-induced hearing loss, the more common type, which is the result of prolonged exposure to intense sound (e.g., noise associated with the workplace and leisure-time activities). The remaining options are not related to noise but rather to the amplitude of the sound. PTS: 1 REF: Page 83 23. What type of necrosis results from ischemia of neurons and glial cells? a. Coagulative c. Caseous b. Liquefactive d. Gangrene ANS: B Liquefactive necrosis commonly results from ischemic injury to neurons and glial cells in the brain. The other types of necrosis are not related to ischemic injuries in the brain. PTS: 1 REF: Page 90 24. What type of necrosis is often associated with pulmonary tuberculosis? a. Bacteriologic c. Liquefactive b. Caseous d. Gangrenous ANS: B Caseous necrosis, which commonly results from tuberculous pulmonary infection, particularly Mycobacterium tuberculosis, is a combination of coagulative and liquefactive necrosis. The other types of necrosis are not observed in pulmonary tuberculosis. PTS: 1 REF: Page 90 25. What type of necrosis is associated with wet gangrene? a. Coagulative c. Caseous b. Liquefactive d. Gangrene ANS: B Wet gangrene develops only when neutrophils invade the site, causing liquefactive necrosis. PTS: 1 REF: Page 91 26. Current research supports the believe that, after heart muscle injury, the damage: a. Remains indefinitely because cardiac cells do not reproduce. b. Is repaired by newly matured cardiomyocytes. c. Gradually decreases in size as mitotic cell division occurs. d. Is replaced by hypertrophy of remaining cells. NURSINGTB.COM PATHOPHYSIOLOGY 8TH EDITION MCCANCE TEST BANKNURSINGTB.COM ANS: B The recent discovery that cardiac stem cells exist in the heart and differentiate into various cardiac cell lineages has profoundly changed the understanding of myocardial biology; it is now believed that bone marrow–derived cardiac stem cells or progenitor cells that have the ability to mature into cardiomyocytes may populate the heart after injury. The other options do not accurately describe the process that is believed to occur to address cardiac muscle damage. PTS: 1 REF: Page 52 | What's New box 27. After ovulation, the uterine endometrial cells divide under the influence of estrogen. This process is an example of hormonal: a. Hyperplasia c. Hypertrophy b. Dysplasia d. Anaplasia ANS: A Hormonal hyperplasia chiefly occurs in estrogen-dependent organs, such as the uterus and breast. After ovulation, for example, estrogen stimulates the endometrium to grow and thicken for reception of the fertilized ovum. The other options do not accurately describe the process identified in the question. PTS: 1 REF: Pages 51-53 28. The abnormal proliferation of cells in response to excessive hormonal stimulation is called: a. Dysplasia c. Hyperplasia b. Pathologic dysplasia d. Pathologic hyperplasia ANS: D Pathologic hyperplasia is the abnormal proliferation of normal cells and can occur as a response to excessive hormonal stimulation or the effects of growth factors on target cells (see Figure 2-4). The other options do not accurately identify the term for the results of excessive hormonal stimulation on cells. PTS: 1 REF: Page 53 29. Removal of part of the liver leads to the remaining liver cells undergoing compensatory: a. Atrophy c. Hyperplasia b. Metaplasia d. Dysplasia ANS: C Compensatory hyperplasia is an adaptive mechanism that enables certain organs to regenerate. For example, the removal of part of the liver leads to hyperplasia of the remaining liver cells (hepatocytes) to compensate for the loss. The other options do not accurately identify the compensatory process described in the question. PTS: 1 REF: Pages 52-53 30. What is the single most common cause of cellular injury? a. Hypoxic injury c. Infectious injury b. Chemical injury d. Genetic injury NURSINGTB.COM PATHOPHYSIOLOGY 8TH EDITION MCCANCE TEST BANKNURSINGTB.COM ANS: A Hypoxia, or lack of sufficient oxygen, is the single most common cause of cellular injury (see Figure 2-8). The other options are not a commonly observed as is the correct option. PTS: 1 REF: Page 56 31. During cell injury caused by hypoxia, sodium and water move into the cell because: a. Potassium moves out of the cell, and potassium and sodium are inversely related. b. The pump that transports sodium out of the cell cannot function because of a decrease in ATP levels. c. The osmotic pressure is increased, which pulls additional sodium across the cell membrane. d. Oxygen is not available to bind with sodium to maintain it outside of the cell. ANS: B A reduction in ATP levels causes the plasma membrane’s sodium-potassium (Na+–K+) pump and sodium-calcium exchange to fail, which leads to an intracellular accumulation of sodium and calcium and diffusion of potassium out of the cell. (The Na+–K+ pump is discussed in Chapter 1.) PTS: 1 REF: Page 57 32. In decompression sickness, emboli are formed by bubbles of: a. Oxygen c. Carbon monoxide b. Nitrogen d. Hydrogen ANS: B If water pressure is too rapidly reduced, the gases dissolved in blood bubble out of the solution, forming emboli. Oxygen is quickly redissolved, but nitrogen bubbles may persist and obstruct blood vessels. Ischemia, resulting from gas emboli, causes cellular hypoxia, particularly in the muscles, joints, and tendons, which are especially susceptible to changes in oxygen supply. The remaining options are not involved in the formation of decompression sickness emboli. PTS: 1 REF: Page 77 33. Which is an effect of ionizing radiation exposure? a. Respiratory distress c. DNA aberrations b. Sun intolerance d. Death ANS: C The effects of ionizing radiation may be acute or delayed. Acute effects of high doses, such as skin redness, skin damage, or chromosomal aberrations, occur within hours, days, or months. The delayed effects of low doses may not be evident for years. The other options are not commonly considered effects of radiation exposure. PTS: 1 REF: Pages 78-79 34. What is an example of compensatory hyperplasia? a. Hepatic cells increase cell division after part of the liver is excised. b. Skeletal muscle cells atrophy as a result of paralysis. c. The heart muscle enlarges as a result of hypertension. NURSINGTB.COM PATHOPHYSIOLOGY 8TH EDITION MCCANCE TEST BANKNURSINGTB.COM d. The size of the uterus increases during pregnancy. ANS: A Compensatory hyperplasia is an adaptive mechanism that enables certain organs to regenerate. For example, the removal of part of the liver leads to hyperplasia of the remaining liver cells (hepatocytes) to compensate for the loss. The other options do not accurately describe the term compensatory hyperplasia. PTS: 1 REF: Pages 52-53 35. It is true that nondividing cells are: a. Found in gastrointestinal lining c. Incapable of synthesizing DNA b. Affected by hyperplasia d. Affected by only hypertrophy ANS: A Gastrointestinal lining is made up of rapidly dividing cells. Hyperplasia and hypertrophy take place if the cells are capable of synthesizing DNA; however, only hypertrophy occurs in nondividing cells. PTS: 1 REF: Pages 51-53 36. Dysplasia refers to a(n): a. Abnormal increase in the number of a specific cell type b. True adaptive process at the cellular level c. Modification in the shape of a specific cell type d. Lack of oxygen at the cellular level ANS: C Dysplasia refers only to abnormal changes in the size, shape, and organization of mature cells. PTS: 1 REF: Pages 53-54 37. Current research has determined that chemical-induced cellular injury: a. Affects the permeability of the plasma membrane. b. Is often the result of the damage caused by reactive free radicals. c. Is rarely influenced by lipid peroxidation. d. Seldom involves the cell’s organelles. ANS: B Not all the mechanisms causing chemical-induced membrane destruction are known; however, the only two general mechanisms currently accepted include: (1) direct toxicity by combining with a molecular component of the cell membrane or organelles, and (2) reactive free radicals and lipid peroxidation. PTS: 1 REF: Pages 62-63 If needing more Test Banks, just let me know: [email protected] NURSINGTB.COM PATHOPHYSIOLOGY 8TH EDITION MCCANCE TEST BANKNURSINGTB.COM MULTIPLE RESPONSE 38. Which organs are affected by lead consumption? (Select all that apply.) a. Bones b. Muscles c. Pancreas d. Nerves e. Eyes ANS: A, D The only organ systems provided as options that are primarily affected by lead include the nervous system, bones, kidneys, teeth, cardiovascular, and reproductive and immune systems. PTS: 1 REF: Page 66 39. What effect does fetal alcohol syndrome have on newborns? (Select all that apply.) a. Failure of alveoli to open b. Cognitive impairment c. Incompetent semilunar values d. Esophageal stricture e. Facial anomalies ANS: B, E Fetal alcohol syndrome (FAS) can lead to growth restriction, cognitive impairment, facial anomalies, and ocular disturbances. The other options do not accurately describe the effects of FAS. PTS: 1 REF: Page 69 40. What organs are affected by the type of necrosis that results from either severe ischemia or chemical injury? (Select all that apply.) a. Lungs b. Brain c. Kidneys d. Muscles e. Heart ANS: C, E Coagulative necrosis, which occurs primarily in the kidneys, heart, and adrenal glands, is a common result of hypoxia from severe ischemia or hypoxia caused by chemical injury, especially the ingestion of mercuric chloride. The other options do not accurately identify organs affected by necrosis resulting from ischemia or chemical injury. PTS: 1 REF: Page 90 41. It is true that melanin is: (Select all that apply.) a. Rarely found in epithelial cells b. Found in cells called keratinocytes, which are present in the retina c. A factor in the prevention of certain types of cancer d. Most influential in managing the effects of short-term sunlight exposure NURSINGTB.COM PATHOPHYSIOLOGY 8TH EDITION MCCANCE TEST BANKNURSINGTB.COM e. Accumulated in specific cells found in the skin ANS: B, C, E Melanin accumulates in epithelial cells (keratinocytes) of the skin and retina and is an extremely important pigment because it protects the skin against long exposure to sunlight and is considered an essential factor in the prevention of skin cancer. PTS: 1 REF: Pages 85-86 42. Examples of adaptive cellular responses include: (Select all that apply.) a. Atrophy b. Dysplasia c. Hypertrophy d. Hyperplasia e. Metaplasia ANS: A, C, D, E Atrophy, hypertrophy, hyperplasia, and metaplasia are considered to be adaptive cellular responses. PTS: 1 REF: Page 50 43. Blunt force injuries would include a: (Select all that apply.) a. Bruise to the upper arm, resulting from a fall b. Simple tibia fracture sustained in a skiing accident c. Cut on the finger while slicing vegetables for a salad d. Spleen laceration caused by a punch during a physical fight e. Small caliber gunshot wound to the foot while target shooting ANS: A, B, D Blunt force injuries are the result of tearing, shearing, or crushing types of injuries, resulting in bruises, fractures, and lacerations caused by blows or impacts. Sharp force injuries include cuts. Gunshot wounds require the penetration of the skin and muscle by a bullet. PTS: 1 REF: Page 72 | Table 2-6 44. Which statements are true regarding the effects of marijuana use? (Select all that apply.) a. Smoking the drug results in greater absorption that eating it. b. Heavy use can result in psychomotor impairments. c. Smoking four “joints” a day equals smoking approximately 20 cigarettes. d. Research does not support marijuana use as a factor in developing lung cancer. e. Fetal development appears to be unharmed by marijuana use. ANS: A, B, C NURSINGTB.COM PATHOPHYSIOLOGY 8TH EDITION MCCANCE TEST BANKNURSINGTB.COM With marijuana smoking, approximately 50% of the potent agents are absorbed through the lungs; when marijuana is ingested, however, only 10% is absorbed. With heavy marijuana use, the following adverse effects have been reported: (1) alterations of sensory perceptions, cognitive and psychomotor impairment (e.g., inability to judge time, speed, distance); (2) smoking three or four joints per day is similar to smoking 20 cigarettes per day, in relation to the frequency of chronic bronchitis and may contribute to lung cancer; (3) data from animal studies only, indicate reproductive changes that include reduced fertility, decreased sperm motility, and decreased circulatory testosterone; (4) fetal abnormalities including low birth weight and increased frequency of childhood leukemia; (5) increased frequency of infectious illness, which is thought to be the result of depressed cell-mediated and humoral immunity. PTS: 1 REF: Page 70 | Table 2-5 NURSINGTB.COM PATHOPHYSIOLOGY 8TH EDITION MCCANCE TEST BANKNURSINGTB.COM Chapter 3: The Cellular Environment: Fluids and Electrolytes, Acids and Bases MULTIPLE CHOICE 1. Infants are most susceptible to significant losses in total body water because of an infant’s: a. High body surface–to–body size ratio b. Slow metabolic rate c. Kidneys are not mature enough to counter fluid losses d. Inability to communicate adequately when he or she is thirsty ANS: C Renal mechanisms that regulate fluid and electrolyte conservation are often not mature enough to counter the losses; consequently, dehydration may rapidly develop. Infants can be susceptible to changes in total body water because of their high metabolic rate and the turnover of body fluids caused by their greater body surface area in proportion to their total body size. The inability to communicate their thirst is a problem only when they are poorly cared for. PTS: 1 REF: Page 104 2. Obesity creates a greater risk for dehydration in people because: a. Adipose cells contain little water because fat is water repelling. b. The metabolic rate of obese adults is slower than the rate of lean adults. c. The rate of urine output of obese adults is higher than the rate of output of lean adults. d. The thirst receptors of the hypothalamus do not function effectively. ANS: A The percentage of total body water (TBW) varies with the amount of body fat and age. Because fat is water repelling (hydrophobic), very little water is contained in adipose cells. Individuals with more body fat have proportionately less TBW and tend to be more susceptible to fluid imbalances that cause dehydration. PTS: 1 REF: Page 104 3. A patient’s blood gases reveal the following findings: pH, 7.3; bicarbonate (HCO3) 27 mEq/L; carbon dioxide (CO2), 58 mm Hg. What is the interpretation of these gases? a. Respiratory alkalosis c. Respiratory acidosis b. Metabolic acidosis d. Metabolic alkalosis ANS: C The values provided in this question characterize only acute uncompensated respiratory acidosis. PTS: 1 REF: Pages 129-130 4. Water movement between the intracellular fluid (ICF) compartment and the extracellular fluid (ECF) compartment is primarily a function of: a. Osmotic forces c. Antidiuretic hormone b. Plasma oncotic pressure d. Hydrostatic forces NURSINGTB.COM PATHOPHYSIOLOGY 8TH EDITION MCCANCE TEST BANKNURSINGTB.COM ANS: A The movement of water between the ICF and ECF compartments is primarily a function of osmotic forces. (Osmosis and other mechanisms of passive transport are discussed in Chapter 1.) PTS: 1 REF: Page 105 5. In addition to osmosis, what force is involved in the movement of water between the plasma and interstitial fluid spaces? a. Oncotic pressure c. Net filtration b. Buffering d. Hydrostatic pressure ANS: D Water moves between the plasma and interstitial fluid through the forces of only osmosis and hydrostatic pressure, which occur across the capillary membrane. Buffers are substances that can absorb excessive acid or base to minimize pH fluctuations. Net filtration is a term used to identify fluid movement in relationship to the Starling hypothesis. Oncotic pressure encourages water to cross the barrier of capillaries to enter the circulatory system. PTS: 1 REF: Page 105 6. Venous obstruction is a cause of edema because of an increase in which pressure? a. Capillary hydrostatic c. Capillary oncotic b. Interstitial hydrostatic d. Interstitial oncotic ANS: A Venous obstruction can increase the hydrostatic pressure of fluid in the capillaries enough to cause fluid to escape into the interstitial spaces. The remaining options are not causes of edema resulting from venous obstruction. PTS: 1 REF: Page 106 7. At the arterial end of capillaries, fluid moves from the intravascular space into the interstitial space because the: a. Interstitial hydrostatic pressure is higher than the capillary hydrostatic pressure. b. Capillary hydrostatic pressure is higher than the capillary oncotic pressure. c. Interstitial oncotic pressure is higher than the interstitial hydrostatic pressure. d. Capillary oncotic pressure is lower than the interstitial hydrostatic pressure. ANS: B At the arterial end of capillaries, fluid moves from the intravascular space into the interstitial because capillary hydrostatic pressure is higher than the capillary oncotic pressure. PTS: 1 REF: Page 105 8. Low plasma albumin causes edema as a result of a reduction in which pressure? a. Capillary hydrostatic c. Plasma oncotic b. Interstitial hydrostatic d. Interstitial oncotic ANS: C NURSINGTB.COM PATHOPHYSIOLOGY 8TH EDITION MCCANCE TEST BANKNURSINGTB.COM Losses or diminished production of plasma albumin is the only option that contributes to a decrease in plasma oncotic pressure. PTS: 1 REF: Pages 106-107 9. Secretion of antidiuretic hormone (ADH) and the perception of thirst are stimulated by a(n): a. Decrease in serum sodium c. Increase in glomerular filtration rate b. Increase in plasma osmolality d. Decrease in osmoreceptor stimulation ANS: B Secretion of ADH and the perception of thirst are primary factors in the regulation of water balance. Thirst is a sensation that stimulates water-drinking behavior. Thirst is experienced when water loss equals 2% of an individual’s body weight or when osmotic pressure increases. The other options do not accurately describe how ADH and the perception of thirst are related. PTS: 1 REF: Page 109 10. Thirst activates osmoreceptors by an increase in which blood plasma? a. Antidiuretic hormone c. Hydrostatic pressure b. Aldosterone d. Osmotic pressure ANS: D Thirst is experienced when water loss equals 2% of an individual’s body weight or when osmotic pressure increases. Dry mouth, hyperosmolality, and plasma volume depletion activate osmoreceptors (neurons located in the hypothalamus that are stimulated by increased osmotic pressure). The other options do not accurately identify what increases to activate osmoreceptors. PTS: 1 REF: Page 109 11. It is true that natriuretic peptides: a. Decrease blood pressure and increase sodium and water excretion. b. Increase blood pressure and decrease sodium and water excretion. c. Increase heart rate and decrease potassium excretion. d. Decrease heart rate and increase potassium excretion. ANS: A Natriuretic peptides are hormones that include atrial natriuretic peptide (ANP) produced by the myocardial atria, brain natriuretic peptide (BNP) produced by the myocardial ventricles, and urodilatin within the kidney. Natriuretic peptides decrease blood pressure and increase sodium and water excretion. PTS: 1 REF: Page 109 12. When changes in total body water are accompanied by proportional changes in electrolytes, what type of alteration occurs? a. Isotonic c. Hypotonic b. Hypertonic d. Normotonic ANS: A NURSINGTB.COM PATHOPHYSIOLOGY 8TH EDITION MCCANCE TEST BANKNURSINGTB.COM Only isotonic alterations occur when proportional changes in electrolytes and water accompany changes in total body water . PTS: 1 REF: Pages 109-110 13. Which enzyme is secreted by the juxtaglomerular cells of the kidney when circulating blood volume is reduced? a. Angiotensin I c. Aldosterone b. Angiotensin II d. Renin ANS: D When circulating blood volume or blood pressure is reduced, renin, an enzyme secreted by the juxtaglomerular cells of the kidney, is released in response to sympathetic nerve stimulation and decreased perfusion of the renal vasculature. The other options are not released by the situation described in the question. PTS: 1 REF: Pages 108-109 14. What mechanism can cause hypernatremia? a. Syndrome of inappropriate antidiuretic hormone b. Hypersecretion of aldosterone c. Brief bouts of vomiting or diarrhea d. Excessive diuretic therapy ANS: B Hypernatremia occurs because of (1) inadequate free water intake, (2) inappropriate administration of hypertonic saline solution (e.g., sodium bicarbonate for treatment of acidosis during cardiac arrest), (3) high sodium levels as a result of oversecretion of aldosterone (as in primary hyperaldosteronism), or (4) Cushing syndrome (caused by the excess secretion of adrenocorticotropic hormone [ACTH], which also causes increased secretion of aldosterone). The other options do not result in hypernatremia. PTS: 1 REF: Page 111 15. What causes the clinical manifestations of confusion, convulsions, cerebral hemorrhage, and coma in hypernatremia? a. High sodium in the blood vessels pulls water out of the brain cells into the blood vessels, causing brain cells to shrink. b. High sodium in the brain cells pulls water out of the blood vessels into the brain cells, causing them to swell. c. High sodium in the blood vessels pulls potassium out of the brain cells, which slows the synapses in the brain. d. High sodium in the blood vessels draws chloride into the brain cells followed by water, causing the brain cells to swell. ANS: A Hypertonic (hyperosmolar) imbalances result in an extracellular fluid concentration greater than 0.9% salt solution (e.g., water loss or solute gain); cells shrink in a hypertonic fluid (see Table 3-7). This shrinking of cells results in the symptoms described in the question. The other options do not accurately describe the cause of these symptoms as they relate to hypernatremia. NURSINGTB.COM PATHOPHYSIOLOGY 8TH EDITION MCCANCE TEST BANKNURSINGTB.COM PTS: 1 REF: Page 111 16. Vomiting-induced metabolic alkalosis, resulting in the loss of chloride, causes: a. Retained sodium to bind with the chloride b. Hydrogen to move into the cell and exchange with potassium to maintain cation balance c. Retention of bicarbonate to maintain the anion balance d. Hypoventilation to compensate for the metabolic alkalosis ANS: C When vomiting with the depletion of ECF and chloride (hypochloremic metabolic alkalosis) causes acid loss, renal compensation is not effective; the volume depletion and loss of electrolytes (sodium [Na+], potassium [K+], hydrogen [H+], chlorine [Cl–]) stimulate a paradoxic response by the kidneys. The kidneys increase sodium and bicarbonate reabsorption with the excretion of hydrogen. Bicarbonate is reabsorbed to maintain an anionic balance because the ECF chloride concentration is decreased. The other options do not accurately describe the mechanism that results from vomiting-induced metabolic alkalosis. PTS: 1 REF: Page 128 17. The pathophysiologic process of edema is related to which mechanism? a. Sodium depletion b. Decreased capillary hydrostatic pressure c. Increased plasma oncotic pressure d. Lymphatic obstruction ANS: D The pathophysiologic process of edema is related to an increase in the forces favoring fluid filtration from the capillaries or lymphatic channels into the tissues. The most common mechanisms are increased capillary hydrostatic pressure, decreased plasma oncotic pressure, increased capillary membrane permeability and lymphatic obstruction, and sodium retention. PTS: 1 REF: Page 105 18. Insulin is used to treat hyperkalemia because it: a. Stimulates sodium to be removed from the cell in exchange for potassium. b. Binds to potassium to remove it through the kidneys. c. Transports potassium from the blood to the cell along with glucose. d. Breaks down the chemical components of potassium, causing it to be no longer effective. ANS: C Insulin contributes to the regulation of plasma potassium levels by stimulating the Na+, potassium–adenosine triphosphatase (K+–ATPase) pump, thereby promoting the movement of potassium simultaneously into the liver and muscle cells with glucose transport after eating. The intracellular movement of potassium prevents an acute hyperkalemia related to food intake. The other options do not accurately describe how insulin is used to treat hyperkalemia. NURSINGTB.COM PATHOPHYSIOLOGY 8TH EDITION MCCANCE TEST BANKNURSINGTB.COM PTS: 1 REF: Page 114 19. A major determinant of the resting membrane potential necessary for the transmission of nerve impulses is the ratio between: a. Intracellular and extracellular Na+ c. Intracellular Na+ and extracellular K+ b. Intracellular and extracellular K+ d. Intracellular K+ and extracellular Na+ ANS: B The ratio of K+ in the ICF to K+ in the ECF is the major determinant of the resting membrane potential, which is necessary for the transmission and conduction of nerve impulses, for the maintenance of normal cardiac rhythms, and for the skeletal and smooth muscle contraction. This is not true of the other options. PTS: 1 REF: Page 114 20. During acidosis, the body compensates for the increase in serum hydrogen ions by shifting hydrogen ions into the cell in exchange for which electrolyte? a. Oxygen c. Potassium b. Sodium d. Magnesium ANS: C In states of acidosis, hydrogen ions shift into the cells in exchange for intracellular fluid potassium; hyperkalemia and acidosis therefore often occur together. This is not true of the other options. PTS: 1 REF: Page 117 | Pages 126-127 21. Causes of hyperkalemia include: a. Hyperparathyroidism and malnutrition b. Vomiting and diarrhea c. Renal failure and Addison disease d. Hyperaldosteronism and Cushing disease ANS: C Hyperkalemia should be investigated when a history of renal disease, massive trauma, insulin deficiency, Addison disease, use of potassium salt substitutes, or metabolic acidosis exists. The other options are not known to be causes of hyperkalemia. PTS: 1 REF: Page 119 22. In hyperkalemia, what change occurs to the cells’ resting membrane potential? a. Hypopolarization c. Depolarization b. Hyperexcitability d. Repolarization ANS: A If extracellular potassium concentration increases without a significant change in intracellular potassium, then the resting membrane potential becomes more positive (i.e., changes from –90 to –80 mV) and the cell membrane is hypopolarized (i.e., the inside of the cell becomes less negative or partially depolarized [increase excitability]). PTS: 1 REF: Pages 117-118 NURSINGTB.COM PATHOPHYSIOLOGY 8TH EDITION MCCANCE TEST BANKNURSINGTB.COM 23. The calcium and phosphate balance is influenced by which three substances? a. Parathyroid hormone, vasopressin, and vitamin D b. Parathyroid hormone, calcitonin, and vitamin D c. Thyroid hormone, vasopressin, and vitamin A d. Thyroid hormone, calcitonin, and vitamin A ANS: B Three hormones regulate calcium and phosphate balance: parathyroid hormone (PTH), vitamin D, and calcitonin. Vasopressin, thyroid hormone, and vitamin A do not influence calcium and phosphate balance. PTS: 1 REF: Page 119 24. It is true that Kussmaul respirations indicate: a. Anxiety is a cause of respiratory acidosis. b. A compensatory measure is needed to correct metabolic acidosis. c. Diabetic ketoacidosis is the cause of the metabolic acidosis. d. More oxygen is necessary to compensate for respiratory acidosis. ANS: B Deep, rapid respirations (Kussmaul respirations) are indicative of respiratory compensation for metabolic acidosis. The other options are not true. PTS: 1 REF: Page 128 25. Chvostek and Trousseau signs indicate which electrolyte imbalance? a. Hypokalemia c. Hypocalcemia b. Hyperkalemia d. Hypercalcemia ANS: C Two clinical signs of hypocalcemia are the Chvostek sign and Trousseau sign. These clinical signs are not indicative of any of the other options. PTS: 1 REF: Page 120 26. An excessive use of magnesium-containing antacids and aluminum-containing antacids can result in: a. Hypomagnesemia c. Hyponatremia b. Hypophosphatemia d. Hypokalemia ANS: B The most common causes of hypophosphatemia are intestinal malabsorption and increased renal excretion of phosphate. Inadequate absorption is associated with vitamin D deficiency, the use of magnesium and aluminum-containing antacids (which bind with phosphorus), long-term alcohol abuse, and malabsorption syndromes. The excessive use of such antacids will not result in the other options. PTS: 1 REF: Page 121 27. The most common cause of hypermagnesemia is: a. Hepatitis c. Trauma to the hypothalamus NURSINGTB.COM PATHOPHYSIOLOGY 8TH EDITION MCCANCE TEST BANKNURSINGTB.COM b. Renal failure d. Pancreatitis ANS: B Renal failure usually causes hypermagnesemia, in which magnesium concentration is greater than 2.5 mEq/L. Hypermagnesemia is not a result of the other options. PTS: 1 REF: Page 122 28. Physiologic pH is maintained at approximately 7.4 because bicarbonate (HCO3) and carbonic acid (H2CO3) exist in a ratio of: a. 20:1 c. 10:2 b. 1:20 d. 10:5 ANS: A The relationship between HCO3 and H2CO3 is usually expressed as a ratio. When the pH is 7.4, this ratio is 20:1 (HCO3:H2CO3). The other options do not accurately identify physiologic pH by the correct ratio of HCO3 and H2CO3. PTS: 1 REF: Page 124 29. Which arterial pH will initiate the formation of ammonium (NH4) from ammonia (NH3), referred to as academia, in the tubular lumen of the kidney? a. 7.25 c. 7.55 b. 7.35 d. 7.65 ANS: A Pathophysiologic changes in the concentration of hydrogen ion or base in the blood lead to acid-base imbalances. Acidemia is a state in which the pH of arterial blood is less than 7.35. NH3 is produced from glutamine in the epithelial cell and diffuses to the tubular lumen, where it combines with H+ to form NH4. PTS: 1 REF: Page 126 30. Two thirds of the body’s water is found in its: a. Interstitial fluid spaces c. Intracellular fluid compartments b. Vascular system d. Intraocular fluids ANS: C Two thirds of the body’s water is in the intracellular fluid (ICF) compartment, and one third is in the extracellular fluid (ECF) compartment. The two main ECF compartments are the interstitial fluid and the intravascular fluid, which is the blood plasma. Other ECF compartments include the lymph and the transcellular fluids, such as the synovial, intestinal, biliary, hepatic, pancreatic, and cerebrospinal fluids; sweat; urine; and pleural, synovial, peritoneal, pericardial, and intraocular fluids. PTS: 1 REF: Pages 103-104 31. It is true that when insulin is administered: a. The Na+, K+–ATPase pump is turned off. b. Potassium is moved out of muscle cells. c. The liver increases its potassium levels. d. Glucose transport is impaired. NURSINGTB.COM PATHOPHYSIOLOGY 8TH EDITION MCCANCE TEST BANKNURSINGTB.COM ANS: C Insulin contributes to the regulation of plasma potassium levels by stimulating the Na+, K+–ATPase pump, thereby promoting the movement of potassium simultaneously into the liver and muscle cells with glucose transport after eating. The other options do not accurately describe the effect of insulin administration. PTS: 1 REF: Page 114 32. Increased capillary hydrostatic pressure results in edema because of: a. Losses or diminished production of plasma albumin b. Inflammation resulting from an immune response c. Blockage within the lymphatic channel system d. Sodium and water retention ANS: D Increased capillary hydrostatic pressure can result from venous obstruction or sodium and water retention. The other options do not accurately describe the cause of edema related to increased capillary hydrostatic pressure. PTS: 1 REF: Page 106 33. The existence of hyperkalemia is likely to result in which changes to a person’s electrocardiogram (ECG)? a. Flattened U waves c. Depressed ST segments b. Peaked T waves d. Peaked P waves ANS: B Observed ECG changes include peaked T waves, prolonged PR interval, and absent P wave with a widened QRS complex. The other options are not related to hyperkalemia. PTS: 1 REF: Page 118 MULTIPLE RESPONSE 34. Which groups are at risk for fluid imbalance? (Select all that apply.) a. Women b. Infants c. Men d. Obese persons e. Older adults ANS: B, D, E Kidney function, surface area, total body water, and the hydrophobic nature of fat cells all contribute to the increased risk for fluid imbalance among obese individuals, infants, and older adults. Gender alone is not a risk factor for fluid imbalance. PTS: 1 REF: Pages 104-105 35. Dehydration can cause which result? (Select all that apply.) a. Moist mucous membranes b. Weak pulses NURSINGTB.COM PATHOPHYSIOLOGY 8TH EDITION MCCANCE TEST BANKNURSINGTB.COM c. Tachycardia d. Polyuria e. Weight loss ANS: B, C, E Significant water deficit is demonstrated by symptoms of dehydration that include headache, thirst, dry skin and mucous membranes, elevated temperature, weight loss, and decreased or concentrated urine (with the exception of diabetes insipidus). Skin turgor may be normal or decreased. Symptoms of hypovolemia include tachycardia, weak pulses, and postural hypotension. PTS: 1 REF: Page 112 36. Causes of hypocalcemia include: (Select all that apply.) a. Repeated blood administration b. Pancreatitis c. Decreased reabsorption of calcium d. Hyperparathyroidism e. Kidney stones ANS: A, B Blood transfusions are a common cause of hypocalcemia because the citrate solution used in storing whole blood binds with calcium. Pancreatitis causes a release of lipases into soft-tissue spaces; consequently, the free fatty acids that are formed bind calcium, causing a decrease in ionized calcium. The other options are not recognized causes of hypocalcemia. PTS: 1 REF: Page 120 37. The electrolyte imbalance called hyponatremia exhibits which clinical manifestations? (Select all that apply.) a. Headache b. Seizures c. Paranoia d. Confusion e. Lethargy ANS: A, B, D, E Behavioral and neurologic changes characteristic of hyponatremia include lethargy, headache, confusion, apprehension, seizures, and coma. Paranoia is not associated with hyponatremia. PTS: 1 REF: Page 113 38. The electrolyte imbalance hypercalcemia exhibits which clinical manifestations? (Select all that apply.) a. Diarrhea b. Calcium based kidney stones c. ECG showing narrow T waves d. Lethargy e. Bradycardia NURSINGTB.COM PATHOPHYSIOLOGY 8TH EDITION MCCANCE TEST BANKNURSINGTB.COM ANS: B, D, E Fatigue, weakness, lethargy, anorexia, nausea, and constipation are common. Behavioral changes may occur. Impaired renal function frequently develops, and kidney stones form as precipitates of calcium salts. A shortened QT segment and depressed widened T waves also may be observed on the ECG, with bradycardia and varying degrees of heart block. PTS: 1 REF: Pages 120-121 39. The electrolyte imbalance hypokalemia exhibits which clinical manifestations? (Select all that apply.) a. Paralytic ileus b. Sinus bradycardia c. Atrioventricular block d. Dry mucous membranes e. Tetany ANS: A, B, C A variety of dysrhythmias may occur, including sinus bradycardia, atrioventricular block, paroxysmal atrial tachycardia, and paralytic ileus. The other options are not related to hypokalemia. PTS: 1 REF: Pages 116-117 40. A third of the body’s fluid is contained in the extracellular interstitial fluid spaces that include: (Select all that apply.) a. Urine b. Intraocular fluids c. Lymph d. Blood plasma e. Sweat ANS: A, B, C, E Two thirds of the body’s water is in the intracelluarl fluid (ICF) compartment, and one third is in the extracellular fluid (ECF) compartments. The two main ECF compartments are the interstitial fluid and the intravascular fluid, such as the blood plasma. Interstitial ECF compartments include the lymph and the transcellular fluids, such as the synovial, intestinal, biliary, hepatic, pancreatic, and cerebrospinal fluids; sweat; urine; and pleural, synovial, peritoneal, pericardial, and intraocular fluids. PTS: 1 REF: Pages 103-104 41. An imbalance of potassium can produce which dysfunctions? (Select all that apply.) a. Weakness skeletal muscles b. Cardiac dysrhythmias c. Smooth muscle atony d. Visual impairment e. Hearing loss ANS: A, B, C NURSINGTB.COM PATHOPHYSIOLOGY 8TH EDITION MCCANCE TEST BANKNURSINGTB.COM Symptoms of hyperkalemia vary, but common characteristics are muscle weakness or paralysis and dysrhythmias with changes in the ECG. A wide range of metabolic dysfunctions may result from hypokalemia. Neuromuscular excitability is decreased, causing skeletal muscle weakness, smooth muscle atony, and cardiac dysrhythmias. PTS: 1 REF: Pages 116-118 42. Which statements regarding total body water (TBW) are true? (Select all that apply.) a. During childhood, TBW slowly decreases in relationship to body weight. b. Gender has no influence on TBW until old age. c. Men tend to have greater TBW as a result of their muscle mass. d. Estrogen plays a role in female TBW. e. Older adults experience a decrease in TBW as a result of decreased muscle mass. ANS: A, C, D, E During childhood, TBW slowly decreases to 60% to 65% of body weight. At adolescence, the percentage of TBW approaches adult proportions, and gender differences begin to appear. Men eventually have a greater percentage of body water as a function of increasing muscle mass. Women have more body fat and less muscle as a function of estrogens and therefore have less body water. With increasing age, the percentage of TBW declines further still. The decrease is caused, in part, by an increased amount of fat and a decreased amount of muscle, as well as by a reduced ability to regulate sodium and water balance. PTS: 1 REF: Pages 104-105 MATCHING Match the electrolytes with the corresponding descriptions. Terms may be used more than once. ______ A. Sodium ______ B. Chloride ______ C. Potassium ______ D. Magnesium ______ E. Phosphate 43. Regulates osmolality in the extracellular fluid (ECF) space. 44. Is inversely related to HCO3 concentration. 45. Is a major determinant of resting membrane potential. 46. An intracellular metabolic form is adenosine triphosphate (ATP). 47. Changes in hydrogen ion concentration affect this electrolyte. 43. ANS: A PTS: 1 REF: Page 105 MSC: Sodium is the most abundant ECF ion and is responsible for the osmotic balance of the ECF space. Potassium maintains the osmotic balance of the ICF space. 44. ANS: B PTS: 1 REF: Page 109 MSC: Chloride levels are inversely related to HCO3 concentration. 45. ANS: C PTS: 1 REF: Page 114 MSC: The ratio of K+ in the ICF to K+ in the ECF is the major determinant of the resting membrane potential, which is necessary for the transmission and conduction of nerve impulses, for the maintenance of normal cardiac rhythms, and for skeletal and smooth muscle contraction. (Membrane transport and membrane potentials are discussed in Chapter 1.) NURSINGTB.COM PATHOPHYSIOLOGY 8TH EDITION MCCANCE TEST BANKNURSINGTB.COM 46. ANS: E PTS: 1 REF: Page 119 MSC: Phosphate acts as an intracellular and extracellular anion buffer in the regulation of acid-base balance; it provides energy for muscle contraction in the form of ATP. 47. ANS: C PTS: 1 REF: Page 117 MSC: In states of acidosis, hydrogen ions shift into the cells in exchange for ICF potassium; hyperkalemia and acidosis therefore often occur together. NURSINGTB.COM PATHOPHYSIOLOGY 8TH EDITION MCCANCE TEST BANKNURSINGTB.COM Chapter 4: Genes and Genetic Diseases MULTIPLE CHOICE 1. Inserting bone marrow cells into an individual who produces abnormal erythrocytes is an example of what type of therapy? a. Somatic cell c. Genetic engineering b. Germ cell d. Recombinant DNA ANS: A Gene therapy can be applied in two ways. The less controversial approach is somatic cell therapy, which consists of inserting normal genes into the cells of an individual who has a genetic disease. In this approach, a particular tissue, such as bone marrow cells that produce abnormal erythrocytes, is treated. The correct option is the only one that accurately identifies the therapy described in the question. PTS: 1 REF: Page 137 | Box 4-2 2. DNA replication requires the enzyme DNA polymerase to: a. Travel along the single DNA strand, adding the correct nucleotide to the new strand b. Move along the double strand of DNA to unwind the nucleotides of the double helix c. Hold the double strand apart while the correct nucleotides are added to the strand d. Transport the double strand of DNA from the nucleus to the cytoplasm for protein formation ANS: A The DNA polymerase enzyme travels along the single DNA strand, adding the correct nucleotides to the free end of the new strand (see Figure 4-2, B). The correct option is the only one that accurately describes the process involved in DNA replication using DNA polymerase. PTS: 1 REF: Page 137 3. Transcription is best defined as a process by which: a. DNA polymerase binds to the promoter site on ribonucleic acid (RNA). b. RNA directs the synthesis of polypeptides for protein synthesis. c. RNA is synthesized from a DNA template. d. A base pair substitution results in a mutation of the amino acid sequence. ANS: C Transcription is the process by which RNA is synthesized from a DNA template. The correct option is the only one that accurately defines the term transcription. PTS: 1 REF: Page 141 4. The purpose of a staining technique of chromosomes such as Giemsa is to: a. Permit the mitotic process to be followed and monitored for variations. b. Allow for the numbering of chromosomes and the identification of variations. NURSINGTB.COM PATHOPHYSIOLOGY 8TH EDITION MCCANCE TEST BANKNURSINGTB.COM c. Identify new somatic cells formed through mitosis and cytokinesis. d. Distinguish the sex chromosomes from the homologous chromosomes. ANS: B One of the most commonly used stains is Giemsa stain. By using banding techniques, chromosomes can be unambiguously numbered, and individual variation in chromosome composition can be studied. Missing or duplicated portions of chromosomes, which often result in serious diseases, also can be readily identified. The correct option is the only one that accurately describes the purpose of the Giemsa staining technique. PTS: 1 REF: Pages 142-143 5. An amniocentesis indicates a neural tube defect when an increase in which protein is evident? a. Chorionic c. Amniotic b. Alpha fetoprotein d. Embryonic ANS: B Other disorders can be detected with this procedure. These include most neural tube defects, which cause an elevation of alpha fetoprotein in the amniotic fluid, and hundreds of diseases caused by mutations of single genes. The correct option is the only one that accurately identifies the protein responsible for a neural tube defect. PTS: 1 REF: Page 136 | Box 4-1 6. An amniocentesis is recommended for pregnant women who: a. Have a history of chronic illness b. Have a family history of genetic disorders c. Have experienced in vitro fertilization d. Had a late menarche ANS: B Amniocentesis is recommended only for pregnancies known to have an elevated risk for a genetic disease or in women older than 30 to 35 years of age. The correct option is the only one that accurately describes a criterion for ordering an amniocentesis. PTS: 1 REF: Page 136 | Box 4-1 7. The most clinically useful technique for prenatal diagnosis of chromosomal abnormalities at 3 months’ (12 weeks’) gestation is: a. Gene mapping c. Amniocentesis b. Linkage analysis d. Chorionic villus sampling ANS: D Chorionic villus sampling consists of extracting a small amount of villous tissue directly from the chorion. This procedure can be performed at 10 weeks’ gestation and does not require in vitro culturing of cells; sufficient numbers are directly available in the extracted tissue. Thus the procedure allows prenatal diagnosis at approximately 3 months’ gestation rather than at nearly 5 months’ gestation. The correct option is the only one that accurately describes the most useful technique for prenatal diagnosis of chromosomal abnormalities. PTS: 1 REF: Page 136 | Box 4-1 NURSINGTB.COM PATHOPHYSIOLOGY 8TH EDITION MCCANCE TEST BANKNURSINGTB.COM 8. The term for an error in which homologous chromosomes fail to separate during meiosis or mitosis is: a. Aneuploidy c. Polyploidy b. Nondisjunction d. Translocation ANS: B Aneuploidy is usually the result of nondisjunction, an error in which homologous chromosomes or sister chromatids fail to separate normally during meiosis or mitosis. The correct option is the only one that is used to describe an error in chromosomal separation during reproduction. PTS: 1 REF: Page 145 9. Which clinical manifestations would be expected for a child who has complete trisomy of the twenty-first chromosome? a. Widely spaced nipples, reduced carrying angle at the elbow, and sparse body hair b. An IQ of 25 to 70, low nasal bridge, protruding tongue, and flat, low-set ears c. High-pitched voice, tall stature, gynecomastia, and an IQ of 60 to 90 d. Circumoral cyanosis, edema of the feet, short stature, and mental slowness ANS: B Individuals with this disease are mentally retarded, with IQs usually ranging from 25 to 70. The facial appearance is distinctive and exhibits a low nasal bridge, epicanthal folds (which produce a superficially Asian appearance), protruding tongue, and flat, low-set ears. The correct option is the only one that accurately describes the clinical manifestations of the complete trisomy of the twenty-first chromosome. PTS: 1 REF: Pages 146-147 10. What is the most common cause of Down syndrome? a. Paternal nondisjunction c. Maternal nondisjunction b. Maternal translocations d. Paternal translocation ANS: C Nondisjunction during the formation of one of the parent’s gametes or during early embryonic development occurs in approximately 97% of infants born with Down syndrome. In approximately 90% to 95% of infants, the nondisjunction occurs in the formation of the mother’s egg cell. The correct option is the only one that accurately describes the most common cause of Down syndrome. PTS: 1 REF: Page 147 11. What syndrome, characterized by an absent homologous X chromosome with only a single X chromosome, exhibits features that include a short stature, widely spaced nipples, and webbed neck? a. Down c. Turner b. Cri du chat d. Klinefelter ANS: C NURSINGTB.COM PATHOPHYSIOLOGY 8TH EDITION MCCANCE TEST BANKNURSINGTB.COM In Turner syndrome, a sex chromosome is missing, and the person’s total chromosome count is 45. Characteristic signs include short stature, female genitalia, webbed neck, shieldlike chest with underdeveloped breasts and widely spaced nipples, and imperfectly developed ovaries. The correct option is the only one that accurately describes the clinical manifestations described in the question. PTS: 1 REF: Page 147 12. A person with 47, XXY karyotype has the genetic disorder resulting in which syndrome? a. Turner c. Down b. Klinefelter d. Fragile X ANS: B A disorder in the chromosome (47, XXY karyotype) results in a disorder known as Klinefelter syndrome. The correct option is the only one that accurately describes a genetic disorder that exhibits the described genetic configuration. PTS: 1 REF: Page 148 13. What is the chromosomal variation that causes Klinefelter syndrome? a. Nondisjunction of the X chromosome in the father b. Translocation of the X chromosome in the mother c. Nondisjunction of X chromosome in the mother d. Translocation of the Y chromosome in the father ANS: C Nondisjunction of the X chromosomes in the mother causes Klinefelter syndrome in the majority of infants, and the frequency of the disorder rises with maternal age. The correct option is the only one that accurately describes the chromosomal variation characteristic of Klinefelter syndrome. PTS: 1 REF: Page 148 14. What is the second most commonly recognized genetic cause of mental retardation? a. Down syndrome c. Klinefelter syndrome b. Fragile X syndrome d. Turner syndrome ANS: B The fragile X syndrome is the second most common genetic cause of mental retardation (after Down syndrome). The correct option is not observed with enough frequency to be recognized as the second most common cause of mental retardation. PTS: 1 REF: Page 151 15. What is the blood type of a person who is heterozygous, having A and B alleles as codominant? a. A c. O b. B d. AB ANS: D NURSINGTB.COM PATHOPHYSIOLOGY 8TH EDITION MCCANCE TEST BANKNURSINGTB.COM When the heterozygote is distinguishable from both homozygotes, the locus is said to exhibit codominance. An example is the ABO blood group, in which heterozygotes having the A and B alleles express both of them as A and B antigens on their red cells (forming blood group AB). The correct option is the only one that accurately describes codominance. PTS: 1 REF: Page 151 16. A couple has two children diagnosed with an autosomal dominant genetic disease. What is the probability that the next child will have the same genetic disease? a. One sixth c. One third b. One fourth d. One half ANS: D Affected heterozygous individuals transmit the trait to approximately one half of their children; however, because gamete transmission is subject to chance fluctuations, it is possible that all or none of the children of an affected parent may have the trait. Nevertheless, when large numbers of matings of this type are studied, the proportion of affected children closely approach one half. PTS: 1 REF: Pages 152-153 17. When a child inherits a disease that is autosomal recessive, it is inherited from: a. Father c. Both parents b. Mother d. Grandparent ANS: C In most cases of recessive disease, both parents of affected individuals are heterozygous carriers. PTS: 1 REF: Pages 155-156 18. People diagnosed with neurofibromatosis have varying degrees of the condition because of the genetic principle of: a. Penetrance c. Dominance b. Expressivity d. Recessiveness ANS: B Expressivity is the extent of variation in phenotype associated with a particular genotype. If expressivity of a disease is variable, then the penetrance may be complete but the severity of the disease can vary greatly. A well-known example of variable expressivity in an autosomal dominant disease is type 1 neurofibromatosis. The correct option is the only one that accurately describes the presence of varying degrees of symptomatologic characteristics. PTS: 1 REF: Page 154 19. Which genetic disease has been linked to a mutation of the tumor-suppressor gene? a. Hemochromatosis c. Familial breast cancer b. Retinoblastoma d. Hemophilia A ANS: B NURSINGTB.COM PATHOPHYSIOLOGY 8TH EDITION MCCANCE TEST BANKNURSINGTB.COM The gene responsible for retinoblastoma has been mapped to the long arm of chromosome 13, and its DNA sequence has been extensively studied. This gene is known as a tumor-suppressor gene; the normal function of its protein product is to regulate the cell cycle so that cells do not grow uncontrollably. The correct option is the only one that accurately identifies a disease resulting from a mutation of the tumor-suppressor gene. PTS: 1 REF: Page 154 20. Cystic fibrosis is caused by what type of gene? a. X-linked dominant c. Autosomal dominant b. X-linked recessive d. Autosomal recessive ANS: D The most common lethal autosomal recessive disease in white children, cystic fibrosis, occurs in approximately 1 in 2500 births. The correct option is the only one that accurately identifies the gene type responsible for cystic fibrosis. PTS: 1 REF: Page 154 21. Which is an important criterion for discerning autosomal recessive inheritance? a. Consanguinity is sometimes present. b. Females are affected more than males. c. The disease is observed in both the parents, as well as in the siblings. d. On average, one half of the offspring of the carrier will be affected. ANS: A Consanguinity is often an important characteristic of pedigrees for recessive diseases; relatives share a certain proportion of genes received from a common ancestor. The correct option is the only one that accurately identifies a required factor in autosomal recessive genetic inheritance. PTS: 1 REF: Page 155 22. Consanguinity refers to the mating of persons: a. Who are unrelated b. When one has an autosomal dominant disorder c. Having common family relations d. When one has a chromosomal abnormality ANS: C Consanguinity refers to the mating of two related individuals, and the offspring of such matings are said to be inbred. The correct option is the only one that accurately identifies consanguinity as it relates to human mating. PTS: 1 REF: Page 155 23. Males, having only one X chromosome (as is expected), are said to be: a. Homozygous c. Hemizygous b. Heterozygous d. Ambizygous ANS: C NURSINGTB.COM PATHOPHYSIOLOGY 8TH EDITION MCCANCE TEST BANKNURSINGTB.COM Males, having only one X chromosome, are said to be hemizygous for genes on this chromosome. The correct option is the only one that accurately identifies the term for males having only one X chromosome. PTS: 1 REF: Page 156 24. Males are more often affected by which type of genetic disease? a. Sex-linked dominant c. Sex-linked b. Sex-influenced d. Sex-linked recessive ANS: D Males are more frequently affected by X-linked recessive diseases, with the difference becoming more pronounced as the disease becomes rarer. The correct option is the only one that is a characteristic of a male-dominate disease. PTS: 1 REF: Page 156 25. An X-linked recessive disease can skip generations because: a. Females are hemizygous for the X chromosome. b. The disease can be transmitted through female carriers. c. Mothers cannot pass X-linked genes to their sons. d. These diseases need only one copy of the gene in females. ANS: B Skipped generations are often observed in X-linked recessive disease pedigrees because the gene can be transmitted through female carriers. Males are hemizygous for genes on the X chromosome. Fathers cannot pass X-linked genes to their sons. X-linked recessive diseases are observed significantly more often in males than in females, because males need only one copy of the gene to express the disease. PTS: 1 REF: Page 157 26. The presence of a zygote having one chromosome with the normal complement of genes and one with a missing gene is characteristic of which genetic disorder? a. Cri du chat c. Klinefelter syndrome b. Down syndrome d. Turner syndrome ANS: A This description is only accurate for Cri du chat syndrome. PTS: 1 REF: Pages 148-149 27. A child with which genetic disorder has a characteristic cry? a. Down syndrome c. Turner syndrome b. Klinefelter syndrome d. Cri du chat ANS: D Cri du chat, which literally means “cry of the cat,” describes the characteristic cry of the affected child. The correct option is the only one with the characteristic cry. PTS: 1 REF: Page 148 NURSINGTB.COM PATHOPHYSIOLOGY 8TH EDITION MCCANCE TEST BANKNURSINGTB.COM 28. Which statement is true regarding X-linked recessive conditions? a. Such diseases use males as phenotypical carriers. b. These conditions are passed from affected father to all of his female children. c. 25% of an affected individual’s grandsons will be affected. d. Cystic fibrosis is an example of such a condition. ANS: B X-linked recessive conditions are passed from an affected father to all of his daughters, who, as phenotypically normal carriers, transmit it to approximately one half of their sons, who are affected. Cystic fibrosis is an autosomal dominant disease. PTS: 1 REF: Page 158 29. DNA formation occurs in which of the cell’s structures? a. Nucleus c. Organelle b. Cytoplasm d. Membrane ANS: A DNA is formed and replicated only in the cell nucleus. PTS: 1 REF: Pages 135-141 30. What is the risk for the recurrence of autosomal dominant diseases? a. 10% c. 50% b. 30% d. 70% ANS: C The recurrence risk for autosomal dominant diseases is usually 50%. PTS: 1 REF: Pages 152-153 31. An individual’s genetic makeup is referred to as his or her: a. Phenotype c. Heterozygous locus b. Genotype d. Homozygous locus ANS: B An individual’s genotype is his or her genetic makeup. The correct option is the only one that accurately defines a person’s genetic makeup. PTS: 1 REF: Page 151 MULTIPLE RESPONSE 32. Which disorders have similar modes of inheritance? (Select all that apply.) a. Cri du chat syndrome b. Duchenne muscular dystrophy c. Polycystic kidney disease d. Down syndrome e. Becker muscular dystrophy ANS: B, E NURSINGTB.COM PATHOPHYSIOLOGY 8TH EDITION MCCANCE TEST BANKNURSINGTB.COM Becker muscular dystrophy and Duchenne muscular dystrophy are the only options that are X-linked recessive disorders. PTS: 1 REF: Page 158 33. The key to accurate DNA replication depends on which complementary base pairs? (Select all that apply.) a. Adenine with thymine b. Adenine with guanine c. Guanine with cytosine d. Cytosine with thymine e. Guanine with thymine ANS: A, C The consistent pairing of adenine with thymine and of guanine with cytosine, known as complementary base pairing, is the key to accurate DNA replication. The correct options are the only ones that accurately identify complementary base pairs. PTS: 1 REF: Page 137 34. Chromosomal abnormalities are the leading known cause of: (Select all that apply.) a. Mental illness b. Mental retardation c. Fetal miscarriage d. Cardiovascular disease e. Respiratory disorders ANS: B, C Chromosome abnormalities are the leading known cause of mental retardation and miscarriage. The correct options are the only ones that accurately identify disorders that related to chromosomal abnormalities. PTS: 1 REF: Page 143 35. Examples of prenatal diagnostic studies include: (Select all that apply.) a. Chorionic villus sampling (CVS) b. Amniocentesis c. Carrier screening d. Preimplantation genetic diagnosis (PGD) e. Drug-sensitivity testing ANS: A, B, D Prenatal testing is conducted before or during the pregnancy but not once labor begins. Such diagnostic studies include amniocenteses, CVS, and PGD. The correct options are the only ones that are exclusively used during the prenatal period. PTS: 1 REF: Page 136 | Box 4-1 36. The advantage derived from human genome sequencing on genetic disorders focuses on: (Select all that apply.) a. Identification of the mutated gene b. Reversal of the mutation NURSINGTB.COM PATHOPHYSIOLOGY 8TH EDITION MCCANCE TEST BANKNURSINGTB.COM c. Diagnosis of the existing disorder d. Appropriate treatment e. Prevention of the disorder ANS: A, C, D The complete human genome sequence will facilitate gene identification, diagnosis, and disease treatment. The reversal of a mutation is not possible. The mapping has no effect on preventing a genetic disorder. PTS: 1 REF: Page 160 MATCHING Match the genetic terms with the corresponding diseases. Terms may be used more than once. ______ A. Autosomal dominant ______ B. Autosomal recessive ______ C. X-linked dominant ______ D. X-linked recessive 37. Cystic fibrosis 38. Duchenne muscular dystrophy 39. Sickle cell disease 40. Huntington disease 37. ANS: B PTS: 1 REF: Page 154 MSC: Is an important example of an autosomal recessive disease is cystic fibrosis. 38. ANS: D PTS: 1 REF: Page 158 MSC: Is an important example of an X-linked recessive disease is Duchenne muscular dystrophy. 39. ANS: B PTS: 1 REF: Page 136 | Box 4-1 MSC: Is an important example of an autosomal recessive disease is sickle cell disease. 40. ANS: A PTS: 1 REF: Page 154 MSC: Another well-known autosomal dominant disease is Huntington disease. NURSINGTB.COM PATHOPHYSIOLOGY 8TH EDITION MCCANCE TEST BANKNURSINGTB.COM Chapter 5: Genes, Environment-Lifestyle, and Common Diseases MULTIPLE CHOICE 1. The data reporting that sickle cell disease affects approximately 1 in 600 American blacks is an example of which concept? a. Incidence c. Ratio b. Prevalence d. Risk ANS: B Prevalence rate is the proportion of the population affected by a disease at a specific point in time. Thus both the incidence rate and the length of the survival period in affected individuals determine prevalence. The incidence rate is the number of new cases of a disease reported during a specific period (typically 1 year), divided by the number of individuals in the population. A numerical expression representing a part of a larger whole or proportion is considered a ratio. Any factor that increases the chance of disease or injury is considered a risk. PTS: 1 REF: Page 165 2. The ratio of the disease among the exposed population to the disease rate in an unexposed population is referred to as what type of risk? a. Attributable c. Causal b. Contingency d. Relative ANS: D A common measure of the effect of a specific risk factor is the relative risk. Assuming a factor is the cause of a disease, attributable risk is the amount of risk that is due to that factor. A future event or circumstance that is possible but cannot be predicted with certainty is a contingency risk. The probability of the outcome is termed a causal risk factor. PTS: 1 REF: Page 165 3. Empirical risks for most multifactorial diseases are based on: a. Chromosomal testing c. Liability thresholds b. Direct observation d. Relative risks ANS: B For most multifactorial diseases, empirical risks (i.e., risks based on direct observation of data) have been derived. The other options are not the basis for determining the empirical risk of most multifactorial diseases. PTS: 1 REF: Page 167 4. What is the cause of familial hypercholesterolemia (FH)? a. Diet high in saturated fats b. Increased production of cholesterol by the liver c. Reduction in the number of low-density lipoprotein (LDL) receptors on cell surfaces [Show More]

Last updated: 1 year ago

Preview 1 out of 492 pages

Reviews( 0 )

$15.00

Add to cart

Instant download

Can't find what you want? Try our AI powered Search

OR

GET ASSIGNMENT HELP
91
0

Document information


Connected school, study & course


About the document


Uploaded On

Jul 29, 2021

Number of pages

492

Written in

Seller


seller-icon
Cheryshev

Member since 3 years

102 Documents Sold


Additional information

This document has been written for:

Uploaded

Jul 29, 2021

Downloads

 0

Views

 91

Document Keyword Tags

Recommended For You


$15.00
What is Browsegrades

In Browsegrades, a student can earn by offering help to other student. Students can help other students with materials by upploading their notes and earn money.

We are here to help

We're available through e-mail, Twitter, Facebook, and live chat.
 FAQ
 Questions? Leave a message!

Follow us on
 Twitter

Copyright © Browsegrades · High quality services·